Gradient Problem Move From P(-1,1)

  • Thread starter Thread starter Geometrick
  • Start date Start date
  • Tags Tags
    Gradient
Click For Summary
To maintain the same height at point P(-1,1) on the surface z = (y-x^2)^3, the direction of movement must be perpendicular to the gradient of the function. The gradient at this point is actually the zero vector, indicating that any direction will keep the height constant. The initial computation suggesting a direction of v = <-2, 1> is incorrect due to the gradient being zero. Therefore, any movement from P will not change the height. The conclusion is that there is no specific direction needed to maintain the same height at this point.
Geometrick
Messages
42
Reaction score
0

Homework Statement


You are at P(-1,1) on the surface z = (y-x^2)^3. What direction should you move from P so that your height remains the same?


Homework Equations





The Attempt at a Solution



So I basically do not want my height z to change. In this case, I will take a vector perpendicular to grad f(p), a simple computation shows that grad f(p) will be in the direction of <1,2>, so I take my direction v to be v = <-2, 1>.

I'm just wondering if this is correct?
 
Physics news on Phys.org
Geometrick said:

Homework Statement


You are at P(-1,1) on the surface z = (y-x^2)^3. What direction should you move from P so that your height remains the same?


Homework Equations





The Attempt at a Solution



So I basically do not want my height z to change. In this case, I will take a vector perpendicular to grad f(p), a simple computation shows that grad f(p) will be in the direction of <1,2>, so I take my direction v to be v = <-2, 1>.

I'm just wondering if this is correct?
I think you have the basic idea down, but have made a mistake in the "simple" computation. At (-1, 1) the gradient of f is the zero vector. Both components of the gradient have factors of 3(y - x2)2. When x = -1, y = 1, this factor is 3(1 - (-1)2)2, or 0.
 
Question: A clock's minute hand has length 4 and its hour hand has length 3. What is the distance between the tips at the moment when it is increasing most rapidly?(Putnam Exam Question) Answer: Making assumption that both the hands moves at constant angular velocities, the answer is ## \sqrt{7} .## But don't you think this assumption is somewhat doubtful and wrong?

Similar threads

  • · Replies 8 ·
Replies
8
Views
2K
  • · Replies 16 ·
Replies
16
Views
3K
  • · Replies 22 ·
Replies
22
Views
4K
  • · Replies 9 ·
Replies
9
Views
3K
  • · Replies 4 ·
Replies
4
Views
2K
Replies
1
Views
1K
Replies
2
Views
2K
  • · Replies 2 ·
Replies
2
Views
2K
  • · Replies 7 ·
Replies
7
Views
1K
  • · Replies 1 ·
Replies
1
Views
1K